LSAT and Law School Admissions Forum

Get expert LSAT preparation and law school admissions advice from PowerScore Test Preparation.

 Administrator
PowerScore Staff
  • PowerScore Staff
  • Posts: 8917
  • Joined: Feb 02, 2011
|
#45222
Please post your questions below! Thank you!
 janzegwu
  • Posts: 1
  • Joined: Nov 15, 2018
|
#60445
Can someone explain why C is the correct answer? I was in between choosing B and C but ultimately chose B...
 Robert Carroll
PowerScore Staff
  • PowerScore Staff
  • Posts: 1787
  • Joined: Dec 06, 2013
|
#60692
Jan,

The problem with this argument is that the conclusion is too strong, given the premises. You have a situation where the government allowed a protest. That protest had a message the government supports. Well, it's hard to tell whether the government supports freedom of expression in that case, right? A better test case is when the government allows a protest with a message it doesn't support!

I think an analogy may help. Let's say someone refuses to eat any dessert with dinner. Would you say that person had good willpower? It depends, right? What if the only dessert that restaurant had was carrot cake, and the person refusing dessert hates carrot cake? Refusing to eat something you find disgusting doesn't tell me whether a person is very good at sticking to a diet! On the other hand, if the restaurant had that person's favorite dessert, and that person had enough of an appetite to want dessert, refusing that dessert would be an act of supreme willpower. But if carrot cake is all that's available, is it fair to criticize the person for lack of willpower when you're not really sure what would have happened if more desserts had been available?

That's the analogy here. The government may or may not have allowed a protest with a message it opposed. We don't know, because the only protest we have here is one the government supported. We need more info - we need to know if the government would have been principled in allowing a protest with an opposed message. The political scientist is leaping to a conclusion that the only reason the government allowed the protest was because of the favorable message. So...the assumption is that the government definitely would not have allowed a protest it disagreed with.

This is answer choice (C).

Answer choice (B) does not have to be assumed because it's perfectly consistent with the argument that the protest concerned a government function. Perhaps the protest was protesting the diminished power of the government! Then the government might naturally support it.

Robert Carroll
 snowy
  • Posts: 73
  • Joined: Mar 23, 2019
|
#65688
Is D wrong because fear doesn’t have to be accurate?

I did answer C but just want to make sure my reasoning for eliminating my other contender, D, is valid!
 Erik Shum
PowerScore Staff
  • PowerScore Staff
  • Posts: 25
  • Joined: Jul 25, 2019
|
#66916
Hi Snowy,

I am not sure if I understood your question but "fear" of any kind is not required to support the political scientist's argument and is not assumed by the political scientist.

Get the most out of your LSAT Prep Plus subscription.

Analyze and track your performance with our Testing and Analytics Package.